Sei sulla pagina 1di 32

Practice Questions 1

1. A patient has developed congestive heart failure after experiencing his first myocardial infarction. The pulmonary
signs and symptoms the therapist expect to find include:
A. inspiratory wheezing and shortness of breath.
B. crackles and cough.
C. cough productive of thick yellow secretions.
D. crackles and clubbing of the digits.

2. A newborn is 4 weeks premature and is examined at birth using the APGAR test. Based on the following results, the
neonatal therapist suspects neurological complications are likely with an APGAR of:
A. 3 at 10 minutes.
B. 8 at 5 minutes.
C. 9 at 1 minute.
D. 8 at 1 minute.

3. A physical therapist requested that a PTA perform ultrasound on the left shoulder of a patient. During the treatment
session, the patient experienced an electrical shock. The physical therapist would not be responsible for any injury to
the patient if this was the result of:
A. the patient touching the ultrasound machine.
B. failure of the PTA to use a ground fault interrupter.
C. failure of the PTA to use sufficient ultrasound gel.
D. faulty circuitry.

4. A 55 year-old individual with documented coronary artery disease but no history of myocardial infarction is enrolled
in an exercise class that utilizes circuit-interval training. The MOST appropriate spacing of work-rest intervals to
stress the aerobic system is:
A. 1:7
B. 1:10
C. 1:1.5
D. 1:4.5

5. The BEST INITIAL intervention to improve functional mobility in an individual with a stable humeral neck fracture
A. active resistive ROM.
B. isometrics for all shoulder musculature.
C. pendulum exercises.
D. modalities to decrease pain.

6. A dancer with unilateral spondylolysis at L4 is referred for physical therapy. She complains of generalized low back
pain when she stands longer than one hour. Interventions for the subacute phase should include strengthening
exercise for the:
A. abdominals working from neutral to full flexion.
B. multifidi working from full flexion back to neutral.
C. multifidi working from neutral to full extension.
D. abdominals working from full extension to full flexion.

7. A patient has fixed forefoot varus malalignment. Possible compensatory motion(s) or posture(s) might include:
A. genu recurvatum.
B. excessive subtalar pronation.
C. ipsilateral pelvic external rotation.
D. hallux varus.

8. The purpose of ABC Physical Therapy Center is to deliver excellent health care services in a unique, individualized
and participative manner at a reasonable cost. The provision of services under these conditions requires and
encourages creative involvement of physicians, staff, patients and their agents
A. mission statement.
B. values statement.
C. vision statement.
D. strategic plan.

07 Sullivan C
2 Answers

9. A patient presents with weakness of the knee (2/5) resulting from an anterior cruciate ligament injury. The therapists
examination reveals moderate pain (5/10) and excessive translation of the tibia during active knee extension. The
therapist determines functional electrical stimulation (FES) is an appropriate intervention. The protocol for
strengthening the quadriceps and improving stability of the knee should consist of stimulation of the:
A. quadriceps but not the hamstrings.
B. hamstrings immediately before the quadriceps to produce cocontraction.
C. hamstrings but not the quadriceps.
D. quadriceps immediately before the hamstrings to produce cocontraction.

10. A therapist is reviewing a hospital record prior to examining a patient for the first time. The suspected diagnosis is
multiple sclerosis. On the neurologists note, the therapist finds the following: DTR right quadriceps is 2%2b, left
quadriceps is 3%2b. The therapist concludes that:
A. the right DTR is normal while the left is exaggerated.
B. both DTRs are abnormal and indicative of upper motor neuron syndrome.
C. the right DTR is exaggerated while the left is clearly abnormal.
D. both DTRs are abnormal and indicative of hyporeflexia.

11. A 72 year-old patient has an episode of syncope in the physical therapy clinic. The therapist attempts to rule out
orthostatic hypotension as the cause of her fainting. This is BEST done by:
A. checking HR and BP in supine after 5 minutes rest, then repeating in semi-Fowler position.
B. palpating the carotid arteries and taking HR; using the supine position for BP measurements.
C. checking HR and BP at rest, and after 3 and 5 minutes of cycle ergometry exercise.
D. checking resting BP and HR in sitting, then repeating measurements after standing for 1 minute.

12. An inpatient with a grade III diabetic foot ulcer is referred for physical therapy. Panafil has been applied to the
necrotic tissue BID. The wound has no foul smell; however, the therapist notes a green tinge on the dressing. In this
case, the therapist should:
A. document the finding and contact the physician immediately.
B. begin a trial of acetic acid to the wound.
C. document the finding and continue with treatment.
D. fit the patient with a total contact cast.

13. The therapist receives a referral to examine the fall risk in an 82 year-old who lives alone and has had two recent
falls. The activity that represents the MOST common risk factor associated with falls in the elderly is:
A. climbing on a stepstool to reach overhead objects.
B. walking with a roller walker with hand brakes.
C. dressing while sitting on the edge of the bed.
D. turning around and sitting down in a chair.

14. A therapist is performing a prosthetic checkout on a patient with a transfemoral amputation. The prosthesis has been
fitted with a quadrilateral socket. A checkout of the walls of the socket should reveal that the:
A. anterior and lateral walls are 2 inches higher than the posterior and medial walls.
B. posterior and lateral walls are 2 inches higher than the medial and anterior walls.
C. height of the posterior wall is 2 inches less than all the other walls.
D. medial wall is 2 inches higher than the posterior wall while the anterior and lateral walls are the same height.

15. A two month-old child with bilateral hip dislocations is being discharged home from an acute pediatric facility. The
therapist has developed a home exercise program and now needs to instruct the parents. The MOST important item
for the therapist to assess before instructing the parents is:
A. the financial reimbursement plan.
B. their degree of anxiety and attention.
C. their level of formal education.
D. the home environment.

07 Sullivan C
Practice Questions 3

16. The grip strength of a group of 50 to 60 year-olds was investigated. A mean score of 40, SD of 5, and range of 26-57
were reported. The grip strength score for a given patient was determined to be 34. The therapist can safely conclude
that in a normal distribution this patients score fell within:
A. 68%25.
B. 75%25.
C. 95%25.
D. 32%25.

17. Which of the following clinical manifestations is NOT typical of early stage cystic fibrosis?
A. excessive appetite and weight gain.
B. increased pulmonary secretions with airway obstruction.
C. frequent recurrent respiratory infections.
D. salty skin and sweat.

18. An infant demonstrates that the ATNR is NOT obligatory when he/she can turn the head:
A. to either side and open the hand.
B. to one side and look at the extended arm on that side.
C. to one side and bring the opposite hand to mouth.
D. and bring the hand to mouth on the same side.

19. A 75 year-old patient is referred to physical therapy for back pain. Medical tests reveal he is at the end stage of
pancreatic cancer. The physician has told him he has cancer but has chosen not to inform the patient about the
prognosis. If the patient asks the therapist what his prognosis is, the therapists BEST response is to tell him:
A. to discuss his concerns with the physician.
B. to ask the nurse practitioner.
C. everything usually works out for the best.
D. the prognosis.

20. During surgery to remove an apical lung tumor, the long thoracic nerve was injured. Muscle weakness is 3%2b/5. It
would be best to implement exercises:
A. standing using hand weights.
B. supine using weights.
C. supine using a pulley.
D. standing while performing wall push-ups.

21. A physical therapist examination reveals: iliac crests high on the left; PSIS low and posterior on the left; ASIS high
and anterior on the left; standing flexion test shows that the left PSIS moves first and farthest superiorly; Gillets test
demonstrates the left PSIS moves inferiorly and laterally less than right; long sitting test shows the left malleolus
moves short to long; sitting flexion test is negative. In light of the above findings, the therapists diagnosis is:
A. left anterior rotated innominate.
B. left posterior rotated innominate.
C. left upslip.
D. iliac inflare on the left.

22. The highest level of provider risk in reimbursement is related to:


A. the cost-based payment method.
B. the fee-for-service payment method.
C. the capitation payment method.
D. the per diem payment method.

23. A patient presents with partial and full thickness burns on the chest and neck region. The therapist decides to apply
TENS prior to debridement to modulate pain. Which TENS mode should provide the BEST relief?
A. conventional (high rate) TENS.
B. acupuncture-like (low rate) TENS.
C. modulated TENS.
D. brief intense TENS.

07 Sullivan C
4 Answers

24. A patient recovering from traumatic brain injury demonstrates instability during feeding while sitting in a wheelchair.
The therapist determines modification is necessary to ensure optimal function. The FIRST body segment or segments
the therapist would align is the:
A. pelvis.
B. head.
C. lower extremities.
D. trunk.

25. A patient demonstrates beginning recovery Stage 4 movements following a left CVA. The PNF pattern that represents
the BEST choice to promote continued recovery of the right upper extremity through the use of out-of-synergy
movements is:
A. chop, reverse chop with right arm leading.
B. bilateral symmetrical D1 thrust and reverse thrust.
C. lift, reverse lift with right arm leading.
D. bilateral symmetrical D2F and D2E, elbows straight.

26. A 50 year-old individual has limited endurance as a result of a sedentary lifestyle. There is no history of
cardiorespiratory problems. Following an exercise tolerance test, which was negative, appropriate INITIAL exercise
prescription parameters for this individual would be:
A. 30 - 60%25 HRmax
B. 60 - 90%25 HRmax
C. 40 - 60%25 HRmax
D. 45 - 55%25 of VO2max

27. A patient is admitted to a coronary care unit with a mild myocardial infarction, After 2 days the patient is referred to
physical therapy for exercise conditioning. During an initial exercise session on the unit, he reports chest pain,
appears anxious, and wants to go back to bed to rest. The therapists BEST course of action is to terminate the
exercise and:
A. have him sit down and continue to monitor his vital signs carefully.
B. monitor his vital signs and contact his doctor immediately.
C. assist the patient back to bed and contact the charge nurse on his floor.
D. assign the physical therapist assistant to assist him back to his bed and monitor his vital signs carefully.

28. A patient with a 10-year history of scleroderma is referred for physical therapy to improve functional status and
endurance. The patient was recently treated with corticosteroids for a bout of myositis. Examination findings reveal
limited ROM and fibrotic soft tissue along with hyperesthesia. The BEST choice for initial intervention is:
A. soft tissue mobilization and stretching.
B. closed chain and modified aerobic step exercises.
C. treadmill walking using body weight support at an intensity of 40%25 HRmax.
D. AROM exercises and walking in a therapeutic pool.

29. A patient is referred to physical therapy following a fall injury (fractured left hip with ORIF). Medical history reveals
a diagnosis of Stage I Alzheimers disease. At this stage, the behaviors the therapist would NOT expect to find are:
A. anxiety and irritability.
B. language comprehension problems.
C. fragmented memory.
D. restlessness and sundowning.

30. A patient recovering from stroke with minimal lower extremity weakness and spasticity is able to walk without an
assistive device. The therapist observes that as he walks he hikes his pelvis on the affected side during the swing
phase. The BEST initial intervention is:
A. partial wall squats using a small ball held between the knees.
B. standing, marching with manual pressure applied downward on the pelvis.
C. marching while sitting on a therapy ball.
D. bridging exercises progressing to sit-to-stand training.

07 Sullivan C
Practice Questions 5

31. The therapist is instructing a patient with traumatic brain injury how to lock the brakes on his wheelchair. He is right-
handed and his right upper extremity is more affected than his left. To obtain optimal results, the BEST training
strategy is to:
A. guide his right hand through the locking motions, then his left.
B. verbally talk him through the locking motions practicing with both hands simultaneously.
C. have him practice locking the brakes first with his left hand, and then his right.
D. have him practice brake locking using his left hand to assist his right.

32. A physical therapist wants to examine the effects of PNF using the technique of contract-relax on shoulder ROM. A
group of 10 patients with adhesive capsulitis were recruited. A matched group of patients were given straight plane
active-assisted exercise for the same length of time (3 times/week for 6 weeks). In this study the independent variable
is:
A. active-assisted exercise.
B. PNF contract-relax technique.
C. ROM.
D. adhesive capsulitis.

33. A therapist is examining a patient with a 12-year history of emphysema. The clinical finding the therapist would NOT
expect to find is:
A. cor pulmonale.
B. clubbing.
C. cyanosis.
D. decreased A-P to lateral chest ratio.

34. A patient is receiving physical therapy following an ACL repair. After 4 weeks of treatment the patient still complains
of pain and instability even though he reports he has been faithful with his home exercise program and wearing his
orthosis. The original referral was for 7 to 8 weeks of physical therapy. The therapists BEST course of action is to:
A. discontinue the treatment; and discuss the situation with the patient and his referring physician.
B. recommend to the patient that he get a second medical opinion as the surgery seems unsuccessful.
C. complete the full 8 weeks of treatment and carefully document his lack of improvement to ensure full insurance
coverage.
D. re-examine the patient and implement a modified therapy program.

35. During a postural screen for chronic shoulder pain in a recreational swimmer, the therapist observes excessive
internal rotation of the shoulders and winging of the scapula during overhead motion. Intervention should focus on:
A. strengthening of pectoral muscles and stretching of upper trapezius.
B. strengthening of upper trapezius and stretching of pectoral muscles.
C. strengthening middle and lower trapezius and stretching of pectoral muscles.
D. strengthening of rhomboids and stretching of upper trapezius.

36. Re-examination of a patient with a dermal ulcer over the coccyx reveals a wound exposing the deep fascia. There is
no necrotic tissue, exudate is minimal, and the borders of the ulcer are diffusely covered with granulation tissue.
Previous treatment has included daily whirlpool and wet-to-dry dressings with normal saline. Based on the re-
examination, intervention should consist of:
A. continuation of the same treatments.
B. whirlpool and hydrogel dressings.
C. calcium alginate dressings.
D. wound irrigation with pressures below 15 psi.

37. The spinal defect shown in the diagram should be managed with avoidance of lumbar spinal:
A. flexion.
B. rotation.
C. extension.
D. lateral flexion.

07 Sullivan C
6 Answers

38. A computer specialist is unable to work because of weakness and altered sensation in her dominant right hand. She
complains of pain and tingling of the thumb, index finger, long finger, and radial half of the ring finger. The therapist
observes thenar weakness and atrophy. Strength, reflexes, and sensation are within normal limits throughout the
remainder of the right upper extremity. Her signs and symptoms are characteristic of:
A. pronator teres syndrome.
B. cervical root compression.
C. ulnar nerve compression.
D. carpal tunnel syndrome.

39. A patient presents with rapidly progressive symmetrical weakness that started in the distal lower extremity muscles
but now has ascended to include proximal trunk and upper extremity muscles. The motor segments of the lower
cranial nerves are also showing impairment. The patient complains of abnormal sensations of tingling and burning of
the affected extremities. Consciousness, cognition, and communication are all normal. These signs and symptoms are
characteristic of:
A. post-polio syndrome.
B. Guillain-Barr syndrome.
C. multiple sclerosis.
D. amyotrophic lateral sclerosis.

40. A patient recovering from surgery for triple coronary artery bypass grafts is scheduled to begin a Phase III cardiac
rehabilitation program. During the resistance training portion of the circuit training program, the therapist instructs
the patient to AVOID the Valsalva maneuver because:
A. heart rate and blood pressure are likely to be elevated.
B. slowing of pulse and increased venous pressure are possible.
C. a cholinergic or vagal response can occur.
D. the decreased return of blood to the heart can lead to pitting edema.

41. A patient experiences color changes in the skin during position changes of the foot. During elevation, pallor develops.
When the limb is then positioned in the seated hanging position, hyperemia develops. These changes are indicative
of:
A. chronic venous insufficiency.
B. lymphedema.
C. arterial insufficiency.
D. deep vein thrombophlebitis.

42. A 73 year-old is referred to physical therapy for an examination of balance. He has a recent history of falls (two in
the last 6 months). Based on knowledge of balance changes in the elderly and scoring of standardized balance
measures, the test data that BEST indicates increased fall risk is:
A. Functional Reach of 7 inches.
B. Berg Balance score of 50.
C. Tinetti Performance Oriented Mobility Assessment (POMA) score of 27.
D. Timed Get Up %26 Go test result of 13 seconds.

43. A patient walks with a Trendelenburg gait. The MOST appropriate intervention to correct this problem is:
A. standing, stepping with the weaker limb, forward and backward.
B. bridging, holding with Theraband around both thighs.
C. half kneeling, weight shifting onto the weak side (foot).
D. supine, lateral leg slides.

44. A 72 year-old patient is walking on a treadmill in the physical therapy department while his vital signs are being
monitored. It is noted that his SaO2 drops from 97%25 to 95%25. In this case, it would be BEST to:
A. place a 100%25 O2 face mask on the patient for the remainder of the exercise session.
B. place a 40%25 O2 face mask on the patient for the remainder of the exercise session.
C. place 2 liters of O2 by nasal cannula on the patient for the remainder of the exercise session.
D. not use supplemental O2.

07 Sullivan C
Practice Questions 7

45. The therapist is evaluating the needs of a 6 year-old child who is diagnosed with myelodysplasia at the T10 level. The
therapist determines the most appropriate mobility device for this child to use in the school environment is a:
A. bilateral HKAFO.
B. lightweight wheelchair.
C. bilateral KAFO.
D. parapodium.

46. A patient has limited right rotation caused by left thoracic facet joint capsular tightness. The intervention that would
best facilitate improved right rotation in sitting is:
A. trunk flexion with left rotation.
B. trunk flexion with right rotation.
C. trunk extension with right rotation.
D. trunk extension with left rotation.

47. EMG activity in the lower extremities during erect standing is continuous in the:
A. anterior tibialis and peroneals.
B. posterior tibialis and intrinsic foot muscles.
C. quadriceps femoris and anterior tibialis.
D. soleus and gastrocnemius.

48. A patient falls while walking in the parallel bars. The therapist is required to fill out an incident report of the event.
Information in the report should include the names of those involved and:
A. the date of the occurrence, a description of the event, and the cause of the fall.
B. a description of the event, where the patient was injured and the corrective actions to be taken.
C. what occurred, when and where it occurred, and witness statements.
D. witness reports and therapists opinion as to the cause.

49. Four days ago, a patient sustained a deep contusion of the right lateral thigh as a result of a blow on the leg by a steel
beam. Following several cryotherapy treatments, the therapist decides to apply ultrasound [US]. The parameters that
are most appropriate in this case are:
A. pulsed US at 1 MHz.
B. continuous US at 1 MHz.
C. continuous US at 3 MHz.
D. pulsed US at 3 MHz.

50. A 99 year-old woman was found unconscious at home. Two days later the therapist examines her in the hospital.
Findings include normal sensation and movement on the right side of the body with impaired sensation (touch,
pressure, proprioception) and paralysis on the left side of the body. The left side of her lower face and her trunk are
similarly impaired. The MOST LIKELY location of the lesion is the:
A. left side of brainstem.
B. left parietal lobe.
C. spinal cord.
D. right parietal lobe.

51. The loss of sensory function in peripheral neuropathy is often among the first noticeable symptoms. If more than one
nerve is involved, the sensory loss typically appears as:
A. bandlike dysesthesias and paresthesias in the hips and thighs.
B. paresthesias affecting primarily the proximal limb segments and trunk.
C. stocking and glove distribution of the lower and upper extremities.
D. allodynia of the feet accompanied by pronounced dorsiflexor weakness.

52. The therapist is supervising a Phase II cardiac rehabilitation class of 10 patients. One of the patients, who is being
monitored with radiotelemetry, is having difficulty. The therapist decides to terminate the patients exercise session
upon observing:
A. an increase in HR 20 BPM above resting.
B. an increase in systolic BP to 150 and diastolic BP to 90.
C. a 2 AV heart block.
D. 1 mm ST-segment depression, upsloping.

07 Sullivan C
8 Answers

53. A patient presents with severe claudication which is evident when he walks distances greater than 200 feet. He also
exhibits muscle fatigue and cramping of both calf muscles. Upon examination, the therapist finds his skin is pale and
shiny with some trophic nail changes. The BEST choice for intervention is to:
A. begin with an interval walking program, exercising only to the point of pain.
B. avoid any exercise stress until he has been on calcium channel blockers for at least 2 weeks.
C. utilize a walking program of moderate intensity, instructing the patient that some pain is expected and to be
tolerated.
D. utilize non-weightbearing exercises such as cycle ergometry.

54. A 72 year-old is hospitalized with diabetes and a large stage II plantar ulcer located over his right heel. He has been
non-weightbearing for the past 2 weeks as a result of the ulcer. The BEST intervention is:
A. clean and bandage with a sterile gauze dressing.
B. request a surgical consult as available wound dressings will not promote healing.
C. clean and debride the wound, and apply a hydrogel dressing.
D. wash the foot and apply skin lubricants followed by a transparent film dressing.

55. The therapist is prescribing a wheelchair for a patient with left hemiplegia. The MOST appropriate feature to include
in this prescription is:
A. detachable arm rests
B. a 17.5 inch seat height.
C. a 20 inch seat height.
D. elevating legrests.

56. A patient who is participating in a cardiac rehab program suddenly collapses and falls to the floor. The physical
therapist is the lone rescuer on site. The therapist checks for a pulse and finds the patient unresponsive. The therapist
calls the emergency response number, and then opens the airway, checks breathing. The patient is not breathing and
has no pulse. The therapist should:
A. give 2 rescue breaths followed by 15 chest compressions, repeating the cycle.
B. give chest compressions only at a rate of 100 per minute.
C. use the AED to shock the patient immediately; if unsuccessful then initiate CPR.
D. give 2 rescue breaths followed by 30 chest compressions, repeating the cycle.

57. A patient who was casted for 3 weeks following a Grade III right ankle sprain has been referred to physical therapy
for mobility exercises. Examination shows a loss of 10 degrees of dorsiflexion. The patient will have the MOST
difficulty in:
A. ambulating barefoot.
B. descending stairs.
C. descending a ramp.
D. ambulating over rough surfaces.

58. The torque output produced in the sitting position during isokinetic exercise involving the hamstrings is:
A. lower than the torque actually generated by the hamstrings.
B. higher due to eccentric assistance of the quadriceps.
C. higher than the torque actually generated by the contracting hamstrings.
D. lower due to resistance of the quadriceps.

59. A manual therapy technique utilized to correct a closing restriction of T5 on T6 is:


A. central P/A pressure at a 60 degree angle on the spinous process of T6 while stabilizing T5.
B. central P/A pressure at a 45 degree angle on the spinous process of T5 while stabilizing T6.
C. unilateral P/A pressure at a 45 degree angle on the right transverse process of T6 while stabilizing T5.
D. unilateral P/A pressure at a 60 degree angle on the left transverse process of T6 while stabilizing T5.

60. A patient presents with an acute onset of vertigo over night. Symptoms worsen with rapid change in head position. If
the head is held still, symptoms subside usually within 1 to 2 minutes. The MOST likely cause of this patients
problem is:
A. benign paroxysmal positional vertigo.
B. bilateral vestibular neuritis.
C. Mnires disease.
D. acoustic neuroma.

07 Sullivan C
Practice Questions 9

61. A patient with a significant history for coronary artery disease tells the therapist he is currently taking atropine with
careful monitoring. Based on knowledge of the effects of this medication the therapist expects:
A. bradycardia at rest and with exercise.
B. increased heart rate and contractility at rest.
C. reduced blood pressure at rest and with exercise.
D. reduced myocardial ischemia and heart rate.

62. A 67 year-old patient recovering from stroke is on warfarin (Coumadin). During his rehabilitation, it would be
important to watch for:
A. edema and dermatitis.
B. palpitations and edema.
C. cellulitis and xeroderma.
D. hematuria and ecchymosis.

63. A 91 year-old patient has reduced vision as a result of bilateral cataracts. Which of the following is NOT an
appropriate intervention for this patient?
A. using high illumination for reading and ADLs.
B. avoiding having him walk ambulation on shiny floor surfaces.
C. highlighting steps with pastel colors of blues and greens.
D. minimizing visual distractions in his the immediate environment.

64. A patient is having difficulty with stair climbing. During ascent, he is able to position his foot on the step above but is
unable to transfer the weight of his body up to the next stair level. The BEST intervention to solve this problem is:
A. hooklying, bridging.
B. standing, partial wall squats.
C. plantigrade, knee flexion with hip extension.
D. standing, side steps.

65. A 72 year-old patient has been hospitalized, on complete bedrest, for 10 days. A physical therapy referral requests
mobilization out-of-bed and ambulation. The patient complains that today his right calf is aching. If he gets up and
moves around he is sure he will feel better. The therapists examination reveals calf tenderness with slight swelling
and warmth. The therapist decides to:
A. ambulate the patient with his support stockings on.
B. postpone ambulation and report the findings immediately.
C. begin with ankle pump exercises in bed and then ambulate.
D. use only AROM exercises with the patient sitting at the edge of the bed.

66. A nine year-old boy with Duchennes muscular dystrophy is referred for homecare. The physical therapist should
BEGIN the examination by:
A. performing a complete motor examination.
B. performing a functional examination using the weeFIM.
C. asking the child and his parents to describe the boys most serious functional limitations.
D. asking the parents to outline the boys past rehabilitation successes.

67. During an examination of an adolescent female who complains of anterior knee pain, the therapist observes that the
lower extremity shows medial femoral torsion and toeing-in position of the feet. The lower extremity position may be
indicative of excessive hip:
A. anteversion.
B. medial/internal rotation.
C. retroversion.
D. lateral/external rotation.

68. A patient with osteoporosis and no fractures complains of increased mid and low back pain during breathing and
other functional activities. The MOST appropriate interventions for this patient include patient education and:
A. trunk extension and abdominal stabilization exercises.
B. trunk rotation and abdominal stabilization exercises.
C. trunk flexion and extension exercises.
D. trunk flexion, and rotation exercises.

07 Sullivan C
10 Answers

69. A single, 22 year-old woman, who is 3 months pregnant, arrives at a therapists private practice complaining of
shoulder and leg pain. She has a black eye and some bruising at the wrists. The state in which the therapist practices
has direct access. An appropriate course of action is:
A. administer massage for bruising, TENS and ice modalities for pain, as indicated by the examination findings.
B. direct the patient to the nearest Ambulatory Care Center for physician evaluation.
C. perform a comprehensive examination and send her to the emergency room along with a copy of the findings.
D. do a comprehensive examination, and if the therapist suspects abuse report the findings to the appropriate
authorities.

70. A 73 year-old patient presents with a stage III decubitus ulcer on the plantar surface of the right foot. After a series of
conservative interventions with limited success, the therapist chooses to apply electrical stimulation for tissue repair.
The electrical current BEST suited in this case is:
A. low volt biphasic pulsed current.
B. high volt monophasic pulsed current.
C. medium frequency burst current.
D. medium frequency beat current.

71. A patient presents with pain radiating down the posterior hip and thigh as a result of a herniated disk in the lumbar
spine. The therapist decides to apply mechanical traction. If the patient can tolerate it, the PREFERRED patient
position is:
A. supine with one knee flexed.
B. supine with both knees flexed.
C. prone with no pillow.
D. prone with pillow under the abdomen.

72. A patient recovering from stroke demonstrates hemiparesis of his right upper extremity and moderate flexion and
extension synergies (flexion stronger than extension). The therapists goal is strengthen the shoulder muscles first,
specifically the deltoid. The BEST choice is to promote:
A. horizontal adduction with elbow extension.
B. horizontal adduction with elbow flexion.
C. abduction with elbow flexion.
D. abduction with elbow extension.

73. A patient recovering from a middle cerebral artery stroke presents with gaze deviation of the eyes. In this type of
stroke the involved eye may deviate toward:
A. the hemiplegic side.
B. up and in.
C. the sound side.
D. down and out.

74. A patient is referred for physical therapy following an exercise tolerance test. The physician reports the test was
positive and had to be terminated at 7 minutes. Based on the therapists knowledge of this procedure, the therapist
expects the patient may have exhibited:
A. ST segment depression from baseline of 3 mm horizontal or downsloping depression.
B. a hypertensive response with a BP of at least 170/95.
C. increasing angina and dyspnea with progressive increases in the treadmill speed and grade.
D. ECG changes from baseline of 1 mm ST-segment elevation.

75. A college soccer player sustained a hyperextension knee injury when kicking the ball with his other lower extremity.
The patient was taken to the emergency room of a local hospital and was diagnosed with knee sprain. He was sent
to physical therapy the next day for aggressive rehabilitation. As part of the examination to determine the type of
treatment plan to implement, the therapist conducts the test shown in the figure. The type of exercise that is indicated
in the acute phase of treatment if a positive test is found includes:
A. agility exercises.
B. closed chain terminal knee extension exercises.
C. open-chain terminal knee extension exercises.
D. plyometric functional exercises.

07 Sullivan C
Practice Questions 11

76. A patient with a grade III diabetic ulcer is being treated with a calcium alginate wound dressing. This type of dressing
can be expected to:
A. absorb exudate and allow rapid moisture evaporation.
B. facilitate autolytic debridement and absorb exudate.
C. provide semirigid support for the limb while maintaining a sterile field.
D. restrict bacteria from the wound while supporting the tissues.

77. The therapist receives a referral to treat a patient with a 5-year history of cirrhosis and Hepatitis B. The therapist
should:
A. use droplet transmission precautions.
B. use contact precautions.
C. avoid direct exposure to blood and body fluids.
D. ask the patient to wear gloves and avoid contact.

78. A patient with a transtibial amputation is learning to walk using a PTB prosthesis. He is having difficulty maintaining
prosthetic stability from heel-strike to footflat. The muscles that are MOST LIKELY weak are the:
A. knee extensors.
B. back extensors.
C. hip flexors.
D. knee flexors.

79. A patient with a history of coronary artery disease and recent myocardial infarction is exercising in an inpatient
Cardiac Rehab Program. As the patient is new, continuous ECG telemetry monitoring is being done. The therapist
observes the following. The BEST course of action is to:
A. have the patient sit down and sent back to the room after a brief rest period.
B. activate the emergency medical response team.
C. have the patient sit down, continue monitoring, and notify the physician immediately.
D. have the patient sit down, rest, then resume the exercise at a lower intensity.

80. A patient recovering from traumatic brain injury is unable to bring the right foot up on the step during stair climbing
training. The BEST training activity is to:
A. practice stair climbing inside the parallel bars using a 3 inch step.
B. practice marching in place.
C. passively bring the foot up and place it on the 7 inch step.
D. strengthen the hip flexors using an isokinetic training device before attempting stair climbing.

81. Which of the following interventions is NOT beneficial for a patient with a right lower lobe viral pneumonia?
A. ambulation activity, monitoring SaO2, HR, RR, and BP.
B. breathing exercises, encouraging right lateral costal expansion.
C. postural drainage, percussion, and shaking to the right lower lobe.
D. teaching an independent exercise program to the patient.

82. A patient suffered a spinal cord injury with a complete injury (ASIA Level A) at T10. It is now three months post-
injury and she is refusing to participate in her functional training program because the major focus is wheelchair
independence. She is sure she is going to walk again. The therapists BEST approach is to:
A. outline realistic short term goals to improve independence while maintaining for the possibility of further recovery.
B. discuss the harmful effects of denial and restrict all discussions to promoting wheelchair independence.
C. refer the patient for psychological counseling and discharge her from P.T.
D. send the patient home for a short time so she will recognize the need for wheelchair training.

83. The therapist is on a home visit, scheduled at lunchtime, visiting an 18 month-old child with moderate developmental
delay. The therapist notices the child and mother are experiencing difficulties with feeding. The child is slumped
down in the highchair and is unsuccessfully attempting to use a raking grasp to lift cereal pieces to her mouth. Both
the child and mother are frustrated. The FIRST intervention should be to:
A. work on desensitizing the gag reflex.
B. recommend the mother return to breast feeding for a few more months.
C. recommend the mother feed the child baby food instead of cereal for a few more months.
D. reposition the child in a proper sitting position using postural supports.

07 Sullivan C
12 Answers

84. A patient is sent to physical therapy with a diagnosis of frozen shoulder. The MOST effective mobilization
technique for restricted shoulder abduction is:
A. posterior glide at 10 degrees of abduction.
B. inferior glide at 55 degrees of abduction.
C. inferior glide at 95 degrees of abduction.
D. lateral glide in neutral position.

85. A patient presents with insidious onset of pain in the jaw that is referred to the head and neck regions. As best as he
can recall, it may be related to biting into something hard. Cervical ROM is limited in flexion by 20 degrees, cervical
lateral flexion limited to the left by 10 degrees. Mandibular depression is 10mm with deviation to the left, protrusion
is 4mm, and lateral deviation is 15mm to right and 6mm to left. Based on these findings the diagnosis for this patient
would be:
A. capsule-ligamentous pattern of TMJ on the left.
B. weak lateral pterygoids on the left.
C. weak lateral pterygoids on the right.
D. cervical spine and TMJ capsular restrictions on the left.

86. A 35 year-old administrative assistant and mother of three is being treated for a Colles fracture. Her husband wants
to look at her medical record. The physical therapist should:
A. give him the chart as he is a spouse and has a right to view the information.
B. not let him look at the chart because he may misinterpret the documentation.
C. let him look at the chart and be available to answer any questions.
D. deny access to the chart unless written permission by his wife is granted.

87. A patient with a recent history of rib fractures suddenly becomes short of breath during secretion removal techniques.
The patient looks panicked and complains of sharp pain in the left chest. A quick screen shows a deviated trachea to
the right among other signs and symptoms. The MOST LIKELY explanation for the above is:
A. angina.
B. pulmonary emboli.
C. pneumothorax.
D. mucous plugging of an airway.

88. A patient presents with supraspinatus tendinitis. After the initial cryotherapy, the therapist decides to apply
ultrasound. To effectively treat the supraspinatus tendon, the therapist would place the shoulder joint in:
A. slight abduction and external rotation.
B. slight abduction and internal rotation.
C. adduction and internal rotation.
D. adduction and external rotation.

89. A patient is 5 weeks post stroke and is demonstrating good recovery of her right upper extremity, characterized as
stage 4 recovery. The BEST choice for a training activity is to have the patient:
A. reach overhead with right arm straight.
B. bear weight on the extended right arm in sitting.
C. put on socks and shoes while in sitting.
D. reach forward to bear weight with the right arm extended against the wall.

90. A patient is 2 days post left CVA and has just been moved from the intensive care unit to a stroke unit. When
beginning the examination, the therapist finds the patients speech slow and hesitant. He is limited to one and two-
word productions and his expressions are awkward and arduous. However, he demonstrates good comprehension. His
difficulties are consistent with:
A. global aphasia.
B. dysarthria.
C. Wernickes aphasia.
D. Brocas aphasia.

07 Sullivan C
Practice Questions 13

91. A patient recovering from traumatic brain injury is functioning at Stage IV on the Rancho Los Amigos Levels of
Cognitive Functioning Scale. During the therapists initial examination the patient becomes agitated and tries to bite
the therapist. The BEST course of action is to:
A. postpone the examination until later in the day when the patient calms down.
B. postpone the examination for one week and then try again.
C. engage in a calming activity and document the behaviors.
D. restructure the formal examination so the therapist can complete it in three very short sessions.

92. A therapist is working on a cardiac care unit in an acute care facility. After exercising a patient recovering from a
ventricular infarct, the therapist notices fatigue and dyspnea after mild activity. Later that day, on a return visit, the
therapist notices the patient has a persistent spasmodic cough while lying in bed. HR is rapid (140) and slight edema
is evident in both ankles. The patient appears anxious and agitated. The therapist suspects:
A. left ventricular failure.
B. right ventricular failure.
C. impending myocardial infarction.
D. developing pericarditis.

93. A marathon runner is examined in physical therapy for anterior pain in the right lower leg. Her resting heart rate is
found to be 46 bpm. The MOST LIKELY explanation for this is that:
A. a low heart rate is suggestive of a hypotensive disorder.
B. a compensatory response to prolonged endurance activity is depressed heart rate with decreased stroke volume.
C. prolonged endurance training has resulted in a low heart rate.
D. incipient coronary pathology is often characterized by an abnormally low heart rate.

94. A patient has been taking corticosteroids (hydrocortisone) for management of adrenocortical insufficiency. She is
referred to physical therapy for mobility training following a prolonged hospitalization. Potential adverse effects that
one can expect from prolonged use of this medication include:
A. confusion and depression.
B. atrophy and osteoporosis.
C. decreased appetite and weight loss.
D. hypotension and myopathy.

95. A therapist is treating a patient with deep partial-thickness burns over 35%25 of the body (chest and arms). Wound
cultures reveal a bacterial count in excess of 105 per gram of tissue on the anterior left arm. The therapist can
reasonably expect:
A. the risk of hypertrophic and keloid scars is low as there is no viable tissue.
B. the burn area is pain free as all nerve endings in the dermal tissue were destroyed.
C. with antibiotics, spontaneous healing can be expected.
D. the infected wound can convert the area to a full-thickness burn.

96. Recently, a 10 year-old patient has begun walking with supination of her foot. With her shoe off, the therapist finds a
new callus on the lateral side of the metatarsal head of the 5th toe. The BEST choice for orthotic prescription is:
A. viscoelastic shoe insert with forefoot medial wedge.
B. scaphoid pad.
C. Thomas heel.
D. viscoelastic shoe insert with forefoot lateral wedge.

97. A 74 year-old patient is recovering from a right total hip replacement (posterolateral incision, cementless fixation).
The MOST appropriate type of bed-to-wheelchair transfer to teach is to have the patient use a:
A. stand-pivot transfer to the surgical side.
B. lateral slide transfer using a transfer board.
C. stand-pivot transfer to the sound side.
D. squat-pivot transfer to the surgical side.

98. A patient with paraplegia at the T10 level wants to participate in wheelchair basketball. He asks the therapist what
options he should look for in a wheelchair. The therapist tells him it would be important to include:
A. a rigid frame.
B. hard-rubber tires.
C. a mid-scapular seat back.

07 Sullivan C
14 Answers

D. a folding frame.
99. The therapist is treating a child with mild developmental delay secondary to 7 weeks prematurity at birth. The child is
now 8 months old and is just learning to sit. The BEST choice for training activity is:
A. standing tilting reactions.
B. sideward protective extension in sitting.
C. prone tilting reactions.
D. supine tilting reactions.

100. A 80 year-old patient with emphysema and a history of hypertension performs a 12 minute walking exercise
tolerance test. He was able to walk 1,106 feet. His vital signs prior to exercise were: HR 104, BP 130/76, SaO2
93%25. At peak exercise his vital signs were: HR 137, BP 162/74, SaO2 92%25. To calculate his exercise intensity
parameters, the BEST method to use is:
A. 40 to 85%25 HR reserve (Karvonens formula).
B. 70 to 80%25 of HRmax
C. 70 to 85%25 of age adjusted predicted HRmax
D. 40 to 50%25 of Max METs.

101. An expected outcome for a fifteen year-old boy with Duchennes muscular dystrophy is:
A. independent in ambulation with no assistive devices.
B. independent in wheelchair mobility.
C. independent in ambulation with Lofstrand crutches.
D. dependent wheelchair mobility.

102. When performing scoliosis screening in a school setting, the optimal age to screen for girls is:
A. 6-8.
B. 12-14.
C. 15-17.
D. 9-11.

103. A patient with a confirmed left C6 nerve root compression due to foraminal encroachment complains of pain in his
left thumb and index finger. The MOST effective cervical position to alleviate this radicular pain in weightbearing is:
A. lower cervical flexion.
B. left sidebending.
C. lower cervical extension.
D. right rotation.

104. A patient is standing with excessive subtalar pronation. Possible correlated motions or postures are:
A. tibial and femoral external rotation, with pelvic external rotation.
B. tibial, femoral, and pelvic internal rotation.
C. tibial, femoral, and pelvic external rotation.
D. tibial and femoral internal rotation with pelvic external rotation.

105. When performing a chart audit, the therapist realizes a date of service was documented inappropriately. The therapist
should:
A. use white out, initial; then add the correct date of service.
B. write over the date with a different color ink, then make and initial the correction.
C. put a single line through the incorrect date, initial, then make and date the correction.
D. erase the incorrect date; then make and date the correction.

106. A therapist is applying high volt pulsed current to the vastus medialis to improve patellar tracking during knee
extension. The patient complains that the current is uncomfortable. To make the current more tolerable to the patient,
yet maintain a good therapeutic effect, the therapist should consider adjusting the:
A. pulse rate.
B. current intensity.
C. pulse duration.
D. current polarity.

07 Sullivan C
Practice Questions 15

107. A patient has a 10-year history of multiple sclerosis and presents with drooping of the right upper eyelid, constriction
of the pupil, and vasodilation with absence of sweating on the face and neck. These signs are characteristic of:
A. Argyll Robertson pupil.
B. homonymous hemianopsia.
C. nystagmus.
D. Horners syndrome.

108. A patient presents with symptoms of uncoordinated eye movements and profound gait and trunk ataxia. He has
difficulty with postural orientation to vertical and tends to tip over even if his eyes are open. Examination of the
extremities reveals little change in tone or coordination. The therapist suspects involvement of the:
A. spinocerebellum.
B. basal ganglia.
C. premotor cortex.
D. vestibulocerebellum.

109. A patient with a 10-year history of diabetes complains of cramping, pain, and fatigue of the right buttock after
walking 400 feet or climbing stairs. When he stops exercising, the pain goes away immediately. The skin of the
involved leg is cool and pale. The therapist checks his medical record and finds no mention of this problem. The
therapist suspects:
A. spinal root impingement.
B. peripheral nerve injury.
C. peripheral arterial disease.
D. Raynauds phenomenon.

110. A patient with low back pain has marked elevation of blood pressure. He complains of mild to severe midabdominal
pain that increases upon exertion. Palpation reveals a pulsing mass in the lower abdomen. The therapist should:
A. instruct in relaxation exercises as a pulsating mass is not unusual with hypertension.
B. discontinue treatment and notify his physician immediately.
C. instruct the patient to contact his physician at the conclusion of therapy.
D. provide hot packs to the abdomen to help relieve the muscle spasm.

111. A patient with a 12-year history of diabetes and a small, purulent tunneling wound located on the left heel is referred
for wound lavage. The therapists BEST choice is to irrigate the wound using:
A. whirlpool with water temperature at 20C.
B. hydrogen peroxide spray.
C. a syringe with Dakins solution while the patient is in the whirlpool.
D. whirlpool with povidone-iodine.

112. An 85 year-old wheelchair dependent resident of a community nursing home has a diagnosis of organic brain
syndrome, Alzheimers type, Stage 2. During the therapists initial interview, the patient demonstrates limited
interaction, mild agitation, and keeps trying to wheel her chair down the hall. As it is late in the day, the therapist
decides to resume the examination the next morning. The patient is most likely exhibiting:
A. frustration because of an inability to communicate.
B. disorientation to time and date.
C. sundowning behavior.
D. inattention as a result of short term memory loss.

113. To increase the stride length of a patient with a right transfemoral amputation who uses a total contact prosthesis, the
therapist should:
A. provide posterior directed resistance to the right ASIS during stance.
B. facilitate the gluteals with tapping over the muscle belly.
C. provide posterior directed resistance to the left ASIS during swing.
D. provide anterior directed resistance to the right PSIS during swing.

114. A patient with a complete C7 spinal cord injury is having difficulty with pushups while in his wheelchair. The MOST
appropriate lead-up activity to enhance wheelchair pushups is:
A. shoulder shrugs.
B. prone-on-elbow pushups.
C. supine bench press using 50%25 one repetition max.

07 Sullivan C
16 Answers

D. supine-on-elbows pushups.

115. A therapist is concerned that a student PT, who is on a final clinical rotation, is having difficulty interacting with the
patients. Specifically, the student does not seem to be willing to listen or demonstrate tolerance and sensitivity to
patient needs. The MOST appropriate conclusion the therapist can reach is that affective objectives for the clinical
education experience are not being met. The primary deficit is:
A. Level 1.0 Receiving.
B. Level 3.0 Valuing.
C. Level 4.0 Organization.
D. Level 5.0 Adherence to a professional code of ethics.

116. A therapist wants to investigate the effectiveness of use of the therapeutic pool for decreasing pain in a group of
patients with fibromyalgia. Two groups of patients were recruited. One group was assigned to exercises and walking
in the pool 3 times/week for 6 months. The other group was assigned to a gym walking program for the same amount
of time. At the end of the study, outcomes were assessed using the McGill Pain Questionnaire and the Health Status
Questionnaire. In order to improve reliability, the lead investigator should:
A. have the same therapist reassess the patients after 6 months.
B. have another therapist reassess after 6 months and compare to normalized scores.
C. utilize a core of 4 experienced therapists to randomly complete all the assessments.
D. perform all the final assessments and compare to the initial assessments performed by a core group of therapists.

117. An apparently healthy individual has many risk factors for coronary artery disease. He is interested in improving his
overall fitness and cardiac health. Following an exercise tolerance test, which was asymptomatic, he is referred for an
exercise class. The most accurate measure of exercise intensity to monitor during his first exercise session is:
A. rating of perceived exertion (RPE).
B. MET level.
C. respiratory rate.
D. heart rate.

118. A mother brings her 8 week-old infant to be examined at Early Intervention because she noticed that the infant was
taking steps in supported standing at two weeks but was not able to do it now. The therapist should:
A. recommend that a full developmental exam be performed by the Early Intervention team.
B. explain that this is normal and that the stepping was a newborn reflex that has gone away.
C. recommend that the mother bring the infant to a pediatric neurologist.
D. explain this was due to a stepping reflex that will re-emerge again around 10 months.

119. A patient complains of pain with mouth opening that makes it difficult for her to eat foods that require chewing.
Examination revealed active mouth opening to be within normal limits of:
A. 15-24mm.
B. 50-64mm.
C. 35-44mm.
D. 65-74mm

120. Correction of flexible forefoot varus with excessive subtalar pronation is BEST accomplished by a customized
orthosis with:
A. medial forefoot and rearfoot valgus posting.
B. lateral forefoot and rearfoot varus posting.
C. rearfoot varus posting only.
D. medial forefoot and rearfoot varus posting.

121. A patient presents with complaints of tingling and paresthesias in the median nerve distribution of the right forearm
and hand. The following tests were found negative bilaterally: Adson, hyperabduction, costoclavicular, Phalens, and
the ulnar nerve Tinel sign. Based on this information, the diagnosis that has NOT been ruled out is:
A. thoracic outlet syndrome.
B. pronator teres syndrome.
C. carpal tunnel syndrome.
D. ulnar nerve entrapment.

07 Sullivan C
Practice Questions 17

122. A patient scheduled for a 30-minute treatment session arrives 10 minutes late. The subsequent patients are also
scheduled for 30 minutes sessions and there is no break in the therapists schedule to accommodate for the patients
tardiness. The therapist should:
A. treat the patient, and bill for the 20 minute session given.
B. send the patient home until the next scheduled appointment, with a request to be punctual.
C. ask the patient if there is a more convenient time for the appointment in order to complete a 30-minute session.
D. treat the patient and bill for 30 minutes of scheduled treatment time.

123. A patient has been referred to the therapist s/p fracture of the femur six months ago. The cast was removed, but the
patient is unable to volitionally contract the quadriceps. The therapist decides to apply electrical stimulation to
stimulate the strengthening of the quadriceps muscle. The BEST choice of electrode size and placement is:
A. large electrodes, closely spaced.
B. small electrodes, closely spaced.
C. small electrodes, widely spaced.
D. large electrodes, widely spaced.

124. A 65 year-old patient is recovering from a right CVA. Due to a series of medical complications she is still bedridden
6 days post-stroke. The therapist wants to reduce the expected negative effects of developing spasticity. The BEST
choice of bed position for this patient is:
A. sidelying on the sound side, affected arm and leg extended at the side, with a pillow between the knees.
B. supine, trunk in midline with small pillow under the scapula, arm extended on supporting pillow, and a small towel
roll under the knee.
C. supine, trunk in slight lateral flexion to the sound side with elbow flexed and supported on a pillow, leg straight.
D. sidelying on the affected side, with the affected shoulder positioned directly underneath, the hip slightly extended
with knee flexed on a pillow.

125. If the subjects vision is blocked either by having the subject close the eyes or by placing a barrier between the part
being tested and the subjects eyes, the therapist can effectively examine:
A. somatosensory integrity.
B. vestibular/visual/somatosensory integration.
C. discriminative touch and fast pain but not proprioception.
D. conscious proprioception but not discriminative touch.

126. A therapist is examining a patient in the coronary intensive care unit. On auscultation the therapist hears an
adventitious S3 heart sound. This finding is indicative of:
A. aortic valve dysfunction.
B. pulmonary valve dysfunction.
C. congestive heart failure.
D. pericarditis.

127. A therapist is asked to advise a healthy 67 year-old individual who wants to take part in a graduated conditioning
program by joining the Mall Walkers Club. The therapists BEST approach to prescribing the intensity of exercise
for this individual is:
A. dyspnea scale.
B. 70-85%25 of maximal age-related HR.
C. HR reserve formula and Ratings of Perceived Exertion.
D. 4-8 MET level walking.

128. A 72 year-old patient recovering from stroke has been using a bilateral exerciser (UBE) to strengthen muscles in his
affected right upper extremity. He is now experiencing burning pain in his shoulder that worsens when his limb is
touched or moved. He also presents with paresthesias and pitting edema in the dorsum of the hand. ROM of the wrist
and fingers is painful and diminished. The therapists BEST course of action is to:
A. discontinue exercise and use ice for pain relief.
B. switch to interval exercise and lower the resistance on the UBE.
C. discontinue UBE exercise, splint the hand and wrist until pain and swelling disappear.
D. discontinue UBE exercise; use massage and active assistive ROM.

07 Sullivan C
18 Answers

129. A 74 year-old patient has had two recent falls coming home from Bingo after dark. Her outside steps are well lit.
Shes unsure why she has fallen but tells the therapist both times she fell just as she came into her house, before she
even had a chance to put her purse down and turn on the inside lights. The therapist suspect a problem with:
A. loss of accommodation and near vision.
B. decreased corneal sensitivity.
C. poor light adaptation.
D. decreased ocular scanning movements.

130. A patient presents with 2/5 muscle strength in both lower extremities and 3/5 strength in the upper extremities. The
MOST appropriate transfer to teach this patient to move from bed to wheelchair is:
A. sliding board.
B. stand pivot.
C. dependent 1-person squat transfer.
D. stand-by assist.

131. A patient with a spinal cord injury at the level of T1 is in the community phase of his mobility training. In order for
him to navigate a standard height curb with his wheelchair, the therapist tells him to:
A. descend backward with the trunk upright and arms hooked around the push handles.
B. ascend backwards with the large wheels first.
C. lift the front casters and ascend in a wheelie position.
D. place the front casters down first during descent.

132. A patient with a long history of systemic steroid use for asthma control has a contraindication for percussion if there
is evidence of:
A. decreased bone density.
B. intercostals muscle wasting.
C. BP > 140/90.
D. barrel chest.

133. A patient presents with a rapid onset of severe weakness of all small muscles of the hands, sharp pleuritic pain in the
shoulder and subscapular area, and a hoarse voice for the past three weeks. She is a hair stylist, has a smoking history
of 22 years, is not on any medications, and has not been ill. Her referral states examine and treat. Based on the above
information this patient is MOST LIKELY exhibiting symptoms of:
A. a C5-6 bilateral foraminal stenosis.
B. thoracic outlet syndrome.
C. ascending Guillain-Barr syndrome.
D. a Pancoast tumor.

134. An examination of a patient reveals the following shoulder signs and symptoms: excessive AROM and PROM; pain
with activity, and on palpation; normal resisted isometric contractions; a positive load/shift test; and negative X-ray
findings. The MOST LIKELY diagnosis is:
A. rotator cuff lesion.
B. impingement.
C. atraumatic shoulder instability.
D. traumatic anterior shoulder dislocation.

135. The therapist is applying cervical traction using a cervical harness. The patient complains of pain in the
temporomandibular joint during the treatment. The therapist should consider:
A. reducing the traction poundage and continuing with the treatment.
B. readjusting the harness and continuing with the treatment.
C. decreasing the treatment time.
D. discontinuing traction.

136. A therapist is treating a patient with a diagnosis of right shoulder rotator cuff tendonitis. The findings of a worksite
ergonomic assessment indicate that the worker is required to perform repetitive reaching activities above shoulder
height. The most appropriate worksite modification would be to:
A. allow the worker to take more frequent rests to avoid overuse.
B. reposition the height of the shelf and items to below shoulder height
C. provide the worker with a taller, sit-stand chair.

07 Sullivan C
Practice Questions 19

D. provide the worker with a standing desk for daily activities.


137. A patient has been diagnosed with impingement syndrome of the shoulder. Following a course of modalities to
control pain and inflammation, progression is to an exercise program to restore normal function of the shoulder. The
BEST PNF diagonal pattern to improve function of the shoulder is:
A. D1 flexion.
B. D2 flexion.
C. D1 extension.
D. D2 extension.

138. A patient presents with pain of the right Achilles tendon as well as on the plantar aspect of the right heel. Pain
developed insidiously and has now lasted several months. On gait analysis the therapist observes abnormal supination
throughout the stance phase of gait. The BEST choice for orthotic intervention is a:
A. cushion heel with a rearfoot valgus post.
B. UCBL insert.
C. flexible shoe insert with forefoot varus post.
D. metatarsal pad.

139. A patient has a 10 year history of peripheral vascular disease affecting the right lower extremity. During auscultation
of the popliteal artery the therapist would NOT expect to find:
A. a bruit.
B. absence of detectable blood flow.
C. 1%2b pulses.
D. 2%2b pulses.

140. A therapist is working in a major medical center and is new to the acute care setting. An orientation session for new
employees concerns infection control. The therapist learns that the most common infection transmitted to healthcare
workers is:
A. hepatitis A.
B. HIV.
C. hepatitis B.
D. tuberculosis.

141. A patient presents with pain, joint swelling, subcutaneous olecranon nodules, and increased erythrocyte
sedimentation rate. These findings are characteristic of:
A. rheumatoid arthritis.
B. fibromyalgia.
C. systemic lupus erythematosus.
D. osteoarthritis.

142. A patient with a transfemoral amputation and an above-knee prosthesis demonstrates knee instability while standing.
His knee buckles easily when he shifts his weight. The therapist suspects the cause of his problem is a:
A. prosthetic knee set too far posterior to the TKA line.
B. prosthetic knee set too far anterior to the TKA line.
C. weak gluteus medius.
D. tight extension aid.

143. A 17 year-old individual with developmental disabilities is referred to a wheelchair clinic for a new wheelchair. She
presents with a severe kyphoscoliosis. The therapist determines the BEST wheelchair modification to order is a:
A. sling seat with dense foam cushion.
B. firm seat back with lateral posture supports and increased seat depth.
C. contoured foam seat.
D. firm seat with lateral knee positioners.

144. A patient is referred for orthotic gait training after receiving a reciprocating gait orthosis. In order for this patient to
walk correctly, it is important to instruct her in the correct sequence. She should shift her weight onto her:
A. crutches and swing both legs through together to a position in front of her crutches.
B. walker and one leg, tuck her pelvis by extending the upper trunk, and swing her other leg through.
C. crutches and swing one leg, then the other forward.
D. walker, extend the upper trunk, and swing both legs forward together to approach the walker.

07 Sullivan C
20 Answers

145. A patient who is undergoing spinal cord rehabilitation is viewed as uncooperative by staff. He refuses to complete the
training activities outlined for him to promote independent functional mobility. A review of his history reveals that
previously he was the director of his own company, with a staff of 20. The MOST appropriate strategy the therapist
can adopt is to:
A. carefully structure the activities and slow down the pace of training.
B. have him work with a supervisor since he works best with people in authority.
C. refer him to another therapist who is male in the hopes that he will have better luck in engaging the patient.
D. involve him in goal setting and have him participate in structuring the training session.

146. A group of 10 patients is recruited into a study investigating the effects of relaxation training on blood pressure. One
group of patients is scheduled to participate in a cardiac rehabilitation program which includes relaxation training 3
times a week for 12 weeks. The other group of patients is instructed to perform activities as usual. At the conclusion
of the study there was no significant difference between the groups; BP decreased significantly in both groups. The
investigator can reasonably conclude:
A. the activities of the non-rehab group were not properly monitored and may account for these results.
B. cardiac rehabilitation is not effective in reducing blood pressure.
C. both groups had blood pressures initially so high that reductions should have been expected.
D. the rehab group was not properly monitored.

147. A patient is using a right KAFO. During orthotic checkout, the therapist discovers the height of the medial upright is
excessive. As she transfers weight to the orthotic leg during gait, the therapist expects that this patient will
demonstrate:
A. posterior trunk bending.
B. lateral lean toward the left.
C. lateral lean toward the right.
D. anterior trunk bending.

148. During gait, a patient with hemiparesis drags his toes during swing. Upon further examination, he has weak
dorsiflexors (able to lift the foot against gravity through 12 range) and a grade of 2 upon examining tone in his
plantar flexors using the Modified Ashworth Scale. An appropriate orthotic modification to correct this problem is:
A. a dorsiflexion assist.
B. spiral AFO.
C. a solid ankle AFO.
D. a dorsiflexion stop.

149. A therapist is examining a 24 month-old child and observes that the child can sit independently, creep in quadruped,
pull-to-stand, cruise sideways, but not walk without support. The therapist concludes that this child is exhibiting:
A. normal cephalocaudal motor development.
B. delay in achieving developmental milestones.
C. normal gross motor development.
D. slow maturation that is within normal limits.

150. Therapist hand/finger placements for posterior to anterior (PA) mobilization techniques to improve down-
gliding/closure of the T7-8 facet joints should be located at the:
A. transverse processes of T8.
B. spinous process of T8.
C. transverse processes of T7.
D. spinous process of T6.

151. A patient complains of waking up several times at night from severe pins and needles in both hands. On
awakening, her hands feel numb for half an hour, and she complains of clumsiness with fine hand movements. The
therapists examination revealed paresthesias in the medial forearm and hypothenar region; reduced grip and pinch
strength; and normal tendon reflexes. Based on the above examination findings the MOST appropriate diagnosis is:
A. thoracic outlet syndrome.
B. ulnar nerve entrapment.
C. carpal tunnel syndrome.
D. pronator teres syndrome.

07 Sullivan C
Practice Questions 21

152. An auto mechanic is referred for physical therapy with a diagnosis of degenerative joint disease affecting C2 and C3.
The patient complains of pain and stiffness in the cervical region and transient dizziness with some cervical motions.
The MOST appropriate INITIAL examination procedure is:
A. Lhermittes test.
B. a vertebral artery test.
C. Oppenheim test.
D. Adsons maneuver.

153. A 22 year-old otherwise healthy patient is recovering from a complete spinal cord injury at the level of L2.
Functional expectations for this patient include:
A. ambulation using bilateral AFOs and canes.
B. ambulation using bilateral KAFOs and a reciprocating walker.
C. ambulation using bilateral KAFOs, crutches and a swing-through gait.
D. wheelchair locomotion using an active duty light weight chair.

154. To examine a patient with a suspected deficit in graphesthesia, the therapist would ask the patient, with eyes shut, to
identify:
A. different weighted, identically shaped cylinders placed in the hand.
B. the vibrations of a tuning fork when placed on a bony prominence.
C. a series of letters traced on the hand.
D. different objects placed in the hand and manipulated.

155. A patient recovering from stroke is ambulatory without an assistive device. He demonstrates a consistent problem
with an elevated and retracted pelvis on the affected side. The BEST therapeutic exercise strategy is to manually
apply:
A. anterior directed pressure during swing.
B. light resistance to forward pelvic rotation during swing.
C. downward compression during stance.
D. light resistance to posterior pelvic elevation during swing.

156. A therapist suspects lower brainstem involvement in a patient with amyotrophic lateral sclerosis. Examination
findings reveal motor impairments of the tongue with ipsilateral wasting and deviation on protrusion. These findings
confirm involvement of cranial nerve:
A. IX.
B. XII.
C. XI.
D. X.

157. A patient is in the intensive care unit following myocardial infarction. Upon examination of the ECG tracings in the
medical record the therapist observes the following changes: ST elevation and T wave inversion in leads II, III, and
AVF. The probable location of the infarct is the:
A. inferior wall.
B. lateral wall.
C. posterior wall.
D. anterior wall.

158. A patient is hospitalized in the ICU with extensive trauma following a motor vehicle accident. A review of her
medical record reveals the following lab values: hematocrit 28%25, hemoglobin 10g/100ml, and serum WBC
12,000/mm. The MOST appropriate conclusion the therapist can reach is:
A. hematocrit and hemoglobin values are abnormal; WBC is normal.
B. only hematocrit values are abnormal.
C. only serum WBC is abnormal.
D. all values are abnormal.

159. A patient with a transfemoral amputation is unable to wear his total contact prosthesis for the past 4 days.
Examination of the residual limb reveals erythema and edema extending over most of the lower anterior limb. He
tells the therapist his limb is very itchy and painful after he scratches it. The MOST LIKELY cause of his symptoms
is:
A. cellulitis.

07 Sullivan C
22 Answers

B. dermatitis.
C. impetigo.
D. herpes zoster.
160. A patient is largely confined to bed and has a stage IV sacral pressure ulcer of three months duration. The BEST
choice of intervention is:
A. a two-inch, convoluted foam mattress.
B. gentle wound cleansing and wet-to-dry gauze dressings.
C. surgical repair.
D. nutritional supplements and pressure relief with a flotation mattress.

161. Following a hip fracture that is now healed, a patient presents with weak hip flexors (2/5). All other muscles are
within functional limits. During gait, the therapist expects that the patient may walk with:
A. forward trunk lean.
B. a circumducted gait.
C. excessive hip flexion.
D. backward trunk lean.

162. A 72 year-old patient with a left transfemoral amputation complains that his left foot is cramping and when he sits it
feels all twisted under him. The therapists BEST choice of intervention is:
A. hot packs and continuous ultrasound to the residual limb.
B. iontophoresis to the distal residual limb using hyaluronidase.
C. appropriate bed positioning with the residual limb in extension.
D. icing and massage to the residual limb.

163. A researcher uses a group of volunteers (healthy, college students) to study the effects of therapy ball exercises on
ankle ROM and balance scores. Twenty volunteers participated in the 20 minute ball exercise class 3 times a week
for 6 weeks. Measurements were taken at the beginning and end of the sessions. Significant differences were found in
both sets of scores and reported at the local PT meeting. Based on this research design, the therapist concludes:
A. therapy ball exercises are an effective intervention to improve ankle stability following chronic ankle sprain.
B. the validity of the study was threatened with the introduction of sampling bias.
C. the reliability of the study was threatened with the introduction of systematic error of measurement.
D. the Hawthorne effect may have influenced the outcomes of the study.

164. A patient has a very large right-sided bacterial pneumonia. Her oxygen level is dangerously low. The body position
that would MOST LIKELY improve her PaO2 is:
A. right sidelying with the head of the bed in the flat position.
B. prone-lying with the head of the bed in the Trendelenburg position.
C. supine-lying with the head of the bed in the Trendelenburg position.
D. left sidelying with the head of the bed in the flat position.

165. An examination of a 46 year-old woman reveals drooping of the shoulder, rotatory winging of the scapula, an
inability to shrug the shoulder, and complaints of aching in the shoulder. Based on these findings, the cause of these
symptoms would MOST LIKELY be due to:
A. muscle imbalance.
B. a lesion of the long thoracic nerve.
C. a lesion of the spinal accessory nerve.
D. strain of the serratus anterior.

166. A patient with spastic hemiplegia is referred to the therapist for ambulation training. The patient is having difficulty
with standing-up from a seated position due to cocontraction of the quadriceps and hamstrings during the knee and
hip extension phase. The therapist wishes to use biofeedback beginning with simple knee extension exercise in the
seated position. The plan is to progress to sit-to-stand training. The initial biofeedback protocol should consist of:
A. high detection sensitivity with recording electrodes placed closely together.
B. low detection sensitivity with recording electrodes placed closely together.
C. low detection sensitivity with recording electrodes placed far apart.
D. high detection sensitivity with recording electrodes placed far apart.

07 Sullivan C
Practice Questions 23

167. Examination of a patient recovering from stroke reveals a loss of pain and temperature sensation on the left side of
the face along with loss of pain and temperature sensation on the right side of the body. All other sensations are
normal. The therapist suspect a lesion in the:
A. midbrain.
B. right cerebral cortex or internal capsule.
C. left posterolateral medulla or pons.
D. left cerebral cortex or internal capsule.

168. A patient is taking the drug Baclofen to control spasticity following spinal cord injury. This medication can be
expected to decrease muscle tone and pain. Adverse reactions of concern to the physical therapist can include:
A. hypertension and palpitations.
B. drowsiness and muscle weakness.
C. headache with visual auras.
D. urinary retention and discomfort.

169. Symptoms of dysdiadochokinesia, dysmetria, and action tremor can be expected with a lesion located in the:
A. neocerebellum.
B. spinocerebellum.
C. vermis.
D. vestibulocerebellum.

170. A patient is exercising in a Phase 3 outpatient cardiac rehabilitation program that utilizes circuit training. One of the
stations utilizes weights. The patient lifts a 5 lb weight, holds it for 20 seconds and then lowers it slowly. The
therapist corrects the activity and tells the patient to reduce the length of the static hold. The static exercise can be
expected to produce:
A. abnormal oxygen uptake.
B. lower heart rate and arterial blood pressure.
C. higher heart rate and arterial blood pressure.
D. reduced normal venous return to the heart and elevated blood pressure.

171. A ten year-old boy with hemophilia fell and injured himself while skateboarding. He was admitted to a pediatric
acute care facility and a therapist sees him that afternoon. Examination reveals a hemarthrosis in his left knee. The
BEST initial intervention for this patient is:
A. instruct the patient in crutch use to protect the joint and assist in early return to walking.
B. a pool program to maintain ROM and strength while he is nonweightbearing.
C. a hot pack for the knee, splint and instruction in nonweightbearing exercises.
D. ice, elevation and a splint for the limb.

172. A 72 year-old male patient recovering from a fractured hip repaired with ORIF has recently been discharged home.
During a home visit, his wife tells the therapist he woke up yesterday morning and told her he couldnt remember
much. Upon examination, the therapist finds some mild motor loss in his right hand and anomia. The therapist
affirms the presence of short-term memory loss. The BEST course of action is to:
A. refer him to his physician as the therapist suspects Alzheimers dementia.
B. refer him to his physician as the therapist suspects a small stroke.
C. advise the family to document and record any new problems they notice over the next week, then report back to
the therapist.
D. ignore the findings as they are expected following surgical anesthesia.

173. A 16 year-old with a 4-year history of Type I diabetes is insulin dependent. He wants to participate in cross country
running. The physical therapist working with the school team advises the athlete to measure his plasma glucose
concentrations before, and after running. In addition, the student should:
A. consume a carbohydrate after practice to avoid hyperglycemia.
B. increase insulin dosage immediately before running.
C. consume a carbohydrate before or during practice to avoid hypoglycemia.
D. avoid carbohydrate-rich snacks within 12 hours of a race

07 Sullivan C
24 Answers

174. A 61 year-old patient with a transtibial amputation has a short residual limb. The BEST choice for prosthetic
replacement is a:
A. supracondylar suspension.
B. supracondylar/suprapatellar suspension.
C. thigh corset.
D. Symes suspension.

175. A physical therapist assistant is ambulating a patient using a three-point crutch gait. The patient is unsteady and
fearful of falling. The patient does not appear to understand the correct gait sequence. The supervising therapists
BEST strategy is to:
A. instruct the PTA to have the patient sit down and utilize mental practice of the task.
B. tell the PTA and patient to stop the ambulation and work on dynamic balance activities instead.
C. instruct the PTA to use a distributed practice schedule to ensure patient success.
D. intervene and teach the correct sequence since the PTA is apparently unable to deal with this special situation.

176. The therapist is reviewing x-rays from a patient with a Grade 2 spondylolisthesis. The BEST radiographic view to
observe this bony anomaly is:
A. posteroanterior.
B. lateral.
C. anteroposterior.
D. oblique.

177. An athlete presents with pain and muscle spasm of the upper back (C7-T8) extending to the lateral border of the
scapula. This encompasses a 10 x 10 cm area on both sides of the spine. If the ultrasound unit only has a 5 cm
soundhead, the therapist should treat:
A. the entire area in five minutes.
B. the entire area in ten minutes.
C. each side allotting two and a half minutes for each section.
D. each side allotting five minutes for each section.

178. A patient with chronic cervical pain is referred to an outpatient physical therapy clinic. Past medical history reveals:
appendectomy, 12 years ago; chronic heart disease; demand-type pacemaker, 8 years ago; whiplash injury, 2 years
ago. Presently the patient complains of pain and muscle spasm in the cervical region. The modality that should NOT
be considered in the case is:
A. mechanical traction.
B. ultrasound.
C. hot pack.
D. transcutaneous electrical stimulation.

179. In posturography, patients who sway more or fall under conditions with the eyes closed and platform moving
(condition 5) or with the visual surround moving and platform moving (condition 6) are likely to demonstrate:
A. problems with sensory selection.
B. vestibular deficiency.
C. somatosensory dependency.
D. visual dependency.

180. A patient presents with severe, frequent seizures originating in the medial temporal lobes. Following bilateral surgical
removal of these areas, he is unable to remember any new information just prior to the surgery to the present. He
cannot recall text he read minutes ago and cannot remember people he has met repeatedly. These outcomes are
indicative of:
A. loss of the hippocampus and declarative memory function.
B. loss of procedural memory and integration with frontal cortex.
C. a primary deficit from the loss of the amygdala.
D. loss of integration of the temporal lobe with the basal ganglia and frontal cortex.

07 Sullivan C
Practice Questions 25

181. A 14 year-old with a body mass index of 30kg/m and a history limited participation in physical activities is referred
for exercise training. The nutritionist has prescribed a diet limiting his caloric intake. The BEST initial exercise
prescription is:
A. 3 weekly sessions of 50 minutes at 70-85%25 VO2max
B. 3 weekly, sessions of 30 minutes at 60-70%25 VO2max
C. 2 weekly, sessions of 60 minutes at 50%25 VO2max
D. 2 daily sessions of 30 minutes at 4070%25 VO2max

182. While on a home visit, a 9 month-old infant becomes unresponsive. The therapist tells the mother to call for
emergency medical services. The therapists IMMEDIATE next step is to tilt the infants head back and give 2 full
breaths, while covering both the mouth and nose. Chest compressions should then be given. The correct procedure
for chest compressions for the infant is to:
A. compress with one hand to a depth of to the depth of the chest at a rate of 100/min.
B. compress lightly with 2 fingers to a depth of to 1 inch at a rate of 100/min.
C. compress lightly with 2 fingers to a depth of 2 inches at a rate of 60/min.
D. compress with both hands to a depth of 1 inch at a rate of 60/min.

183. A patient in an exercise class develops muscle weakness and fatigue. Examination reveals leg cramps and
hyporeflexia. He also experiences frequent episodes of postural hypotension and dizziness. Abnormalities on his
ECG include a flat T wave, prolonged QT interval, and depressed ST segment. The therapist suspects:
A. hyperkalemia
B. hypocalcemia
C. hyponatremia
D. hypokalemia

184. A 64 year-old patient with advanced coronary artery disease and diabetes is receiving functional mobility training in
a P.T. clinic. While walking after lunch, the patient experiences difficulty breathing, starts hyperventilating, and
suffers an episode of syncope. The MOST LIKELY cause of this problem is:
A. postprandial hypertension.
B. coronary artery disease.
C. hyperglycemia.
D. seizures.

185. The MOST appropriate position to guard a patient who is descending stairs for the first time using crutches and non-
weightbearing on the right is to:
A. stand behind and slightly to the right side.
B. stand in front and slightly to the right side.
C. stand in front and slightly to the left side.
D. stand behind and slightly to the right side.

186. A patient with paraplegia at the T10 level is being discharged home. The rehab team is assisting the patients wife in
modifying the home to be barrier-free. A new first floor bedroom and bathroom wing are being built. The builder asks
the therapist about the entrance. The therapist recommends:
A. a ramp with a slope of 1:12 with a level landing at the top of at least 60 by 60 inches.
B. door widths of at least 40 inches with a handrail 34 inches high.
C. door width of 36 inches with an outside door that opens out.
D. a ramp with a grade of 12%25 and a level landing at top of at least 50 by 50 inches.

187. A patient has a 10 year history of Parkinsons disease and has been on levodopa for the past 6 years. He has fallen 3
times in the past month resulting in a Colles fracture. The therapist decides to try postural biofeedback training using
a platform balance training device. The training sequence should focus on:
A. decreasing the limits of stability and improving increasing anterior weight displacement.
B. increasing the limits of stability and improving increasing anterior weight displacement.
C. decreasing the limits of stability and improving increasing posterior weight displacement.
D. increasing the limits of stability and improving center of pressure alignment.

07 Sullivan C
26 Answers

188. A patient is recovering from myocardial infarction and is referred for supervised exercise training. While working out
on a treadmill, the patient begins to develop mild shortness of breath. Upon inspection of the ECG readout the
therapist determines the presence of:
A. sinus rhythm with upsloping ST segment depression.
B. sinus rhythm with SA blocks.
C. sinus rhythm with downsloping ST segment depression.
D. tachycardia with abnormal P waves.

189. A 16 year-old boy with Duchennes muscular dystrophy has been confined to using a power wheelchair for the past
three years and is beginning to develop a 10 degree Cobb angle scoliosis. The BEST strategy to help slow this spinal
curvature at this time would be to:
A. emphasize spinal rotation exercises.
B. alternate the side of the wheelchair power control.
C. emphasize spinal extension exercises.
D. order a new wheelchair with a reclining seat back.

190. A patient presents with a complaint of severe neck and shoulder pain of two days duration. The patient reports falling
asleep on the couch watching TV, and has been stiff and sore since. There is tenderness of the cervical muscles on the
right, with increased pain upon palpation. PROM is most limited in flexion, then sidebending left, and then rotation
left, and active extension. Sidebending right and rotation right is also painful. Based on these examination findings,
the patients diagnosis is:
A. cervical radiculopathy.
B. facet syndrome.
C. cervical strain.
D. herniated disc.

191. A postal worker (mail sorter) complains of numbness and tingling in his right hand in the median nerve distribution.
When the therapist evaluates his work tasks, the therapist notes that he is required to key in the zip codes of about 58
letters per minute. An appropriate administrative control to decrease his exposure would be to:
A. provide the worker with a resting splint to support his wrist.
B. provide him with a height adjustable chair to position his wrists and hands in a neutral alignment.
C. use job rotation during the workday.
D. require the worker to attend a cumulative trauma disorder educational class.

192. A patient with a traumatic brain injury presents with hemiparesis of the left upper extremity. The examination reveals
slight cutaneous and proprioceptive impairment, fair (3/5) strength of the shoulder muscles and triceps and slight
spasticity of the biceps. Voluntary control of the patients left arm has not progressed since admission. The therapist
decides to use functional electrical stimulation and place the active electrode on the triceps to facilitate active
extension of the elbow. The timing sequence BEST to apply is:
A. 2-second ramp up, 5-second stimulation, 2-second ramp down.
B. 5-second ramp up, 5-second stimulation, 5-second ramp down.
C. 2-second ramp up, 10-second stimulation, no ramp down.
D. no ramp up, 10-second stimulation, 2-second ramp down.

193. Following a traumatic brain injury, a patient presents with significant difficulties in learning new skills. He is
wheelchair dependent and will need to learn how to transfer (a skill he has never done before). The BEST strategy to
enhance his motor learning is to:
A. provide consistent feedback using a blocked practice schedule.
B. use only guided movement to ensure correct performance.
C. provide summed feedback after every few trials using a serial practice schedule.
D. provide bandwidth feedback using a random practice schedule.

194. A patient recovering from a CVA presents with predominant involvement of the contralateral lower extremity and
lesser involvement of the contralateral upper extremity. The patient also demonstrates mild apraxia. These clinical
manifestations are characteristic of:
A. middle cerebral artery syndrome.
B. anterior cerebral artery syndrome.
C. posterior cerebral artery syndrome.
D. basilar artery syndrome.

07 Sullivan C
Practice Questions 27

195. A patient presents with significant intermittent claudication with onset after two minutes of walking. On further
examination, the therapist would expect to find:
A. grayish-white appearance of the extremity in both gravity dependent and independent positions.
B. little or no changes in color with changes in extremity position.
C. bright red appearance of the extremity in both gravity dependent and independent positions.
D. elevation-induced pallor and dependent redness with the extremity in the gravity dependent position.

196. Following her cesarean section, a patient tells the therapist that she is anxious to return to her pre-pregnancy level of
physical activity (working out at the gym 3 days a week and running 5 miles every other day). The therapists BEST
advice is to tell her to resume activities with:
A. pelvic floor exercises and refrain from all other exercise and running for at least 6-8 weeks.
B. pelvic floor and gentle abdominal exercises for the first 4-6 weeks.
C. abdominal crunches with return to running after 1 month.
D. a walking program progressing to running after 5 weeks.

197. A 77 year-old patient lives alone and is referred for home physical therapy services to improve functional mobility.
He refuses to get out of his chair. Upon examination, he appears irritable, with poor concentration and memory. He
tells the therapist he has not been sleeping well and has no energy or desire to do anything. He appears anorexic. The
MOST LIKELY explanation for his symptoms is:
A. depression.
B. hypothyroidism.
C. alcohol abuse.
D. pseudodementia.

198. The rehabilitation team is completing a home visit to recommend environmental modifications for a 72 year-old
patient who is scheduled to be discharged next week. He is wheelchair dependent. The bathroom has not been
adapted. Which of the following recommendations is NOT appropriate?
A. adding a tub seat.
B. taking the door off to widen the entrance to 32 inches.
C. adding horizontal grab bars positioned at 45 inches.
D. raising the toilet seat to 18 inches.

199. During gait, the lateral pelvic tilt on the side of the swing leg, observed during frontal plane analysis, serves to:
A. control forward and backward rotations of the pelvis.
B. reduce peak rise of the pelvis.
C. reduce physiological valgum at the knee.
D. reduce knee flexion at mid stance.

200. A patient experiences central thoracic pain while sitting which increases during the day. There are no complaints in
the night or morning. Lifting and carrying heavy objects increase the pain. Based on the above information the
MOST LIKELY diagnosis would be:
A. herniated nucleus pulposus of the spine.
B. spondylodiscitis.
C. thoracic postural syndrome.
D. traumatic compression fracture.

07 Sullivan C
28 Answers

07 Sullivan C
Answers 29

1. crackles and cough.


2. 3 at 10 minutes.
3. faulty circuitry.
4. 1:1.5
5. pendulum exercises.
6. multifidi working from full flexion back to neutral.
7. excessive subtalar pronation.
8. mission statement.
9. hamstrings immediately before the quadriceps to produce cocontraction.
10. the right DTR is normal while the left is exaggerated.
11. checking resting BP and HR in sitting, then repeating measurements after standing for 1 minute.
12. document the finding and continue with treatment.
13. anterior and lateral walls are 2 inches higher than the posterior and medial walls.
14. turning around and sitting down in a chair.
15. their degree of anxiety and attention.
16. 95%25.
17. excessive appetite and weight gain.
18. and bring the hand to mouth on the same side.
19. to discuss his concerns with the physician.
20. standing while performing wall push-ups.
21. left posterior rotated innominate.
22. the capitation payment method.
23. brief intense TENS.
24. pelvis.
25. chop, reverse chop with right arm leading.
26. 60 - 90%25 HRmax
27. have him sit down and continue to monitor his vital signs carefully.
28. AROM exercises and walking in a therapeutic pool.
29. language comprehension problems.
30. marching while sitting on a therapy ball.
31. have him practice locking the brakes first with his left hand, and then his right.
32. PNF contract-relax technique.
33. discontinue the treatment; and discuss the situation with the patient and his referring physician.
34. decreased A-P to lateral chest ratio.
35. strengthening middle and lower trapezius and stretching of pectoral muscles.
36. wound irrigation with pressures below 15 psi.
37. extension.
38. carpal tunnel syndrome.
39. Guillain-Barr syndrome.
40. arterial insufficiency.
41. slowing of pulse and increased venous pressure are possible.
42. Functional Reach of 7 inches.
43. bridging, holding with Theraband around both thighs.
44. not use supplemental O2.
45. lightweight wheelchair.
46. trunk flexion with right rotation.
47. soleus and gastrocnemius.
48. pulsed US at 1 MHz.
49. what occurred, when and where it occurred, and witness statements.
50. right parietal lobe.
51. stocking and glove distribution of the lower and upper extremities.
52. a 2 AV heart block.
53. begin with an interval walking program, exercising only to the point of pain.
54. a 17.5 inch seat height.
55. clean and debride the wound, and apply a hydrogel dressing.
56. give 2 rescue breaths followed by 30 chest compressions, repeating the cycle.
57. descending stairs.
58. higher than the torque actually generated by the contracting hamstrings.
59. central P/A pressure at a 60 degree angle on the spinous process of T6 while stabilizing T5.

07 Sullivan C
30 Answers

60. benign paroxysmal positional vertigo.


61. increased heart rate and contractility at rest.
62. hematuria and ecchymosis.
63. highlighting steps with pastel colors of blues and greens.
64. standing, partial wall squats.
65. postpone ambulation and report the findings immediately.
66. asking the child and his parents to describe the boys most serious functional limitations.
67. anteversion.
68. trunk extension and abdominal stabilization exercises.
69. do a comprehensive examination, and if the therapist suspects abuse report the findings to the appropriate
authorities.
70. high volt monophasic pulsed current.
71. prone with no pillow.
72. abduction with elbow extension.
73. the sound side.
74. ST segment depression from baseline of 3 mm horizontal or downsloping depression.
75. closed chain terminal knee extension exercises.
76. facilitate autolytic debridement and absorb exudate.
77. avoid direct exposure to blood and body fluids.
78. knee extensors.
79. have the patient sit down, continue monitoring, and notify the physician immediately.
80. practice stair climbing inside the parallel bars using a 3 inch step.
81. postural drainage, percussion, and shaking to the right lower lobe.
82. outline realistic short term goals to improve independence while maintaining for the possibility of further
recovery.
83. reposition the child in a proper sitting position using postural supports.
84. inferior glide at 55 degrees of abduction.
85. capsule-ligamentous pattern of TMJ on the left.
86. deny access to the chart unless written permission by his wife is granted.
87. pneumothorax.
88. slight abduction and internal rotation.
89. Brocas aphasia.
90. reach forward to bear weight with the right arm extended against the wall.
91. engage in a calming activity and document the behaviors.
92. left ventricular failure.
93. prolonged endurance training has resulted in a low heart rate.
94. atrophy and osteoporosis.
95. the infected wound can convert the area to a full-thickness burn.
96. viscoelastic shoe insert with forefoot lateral wedge.
97. stand-pivot transfer to the sound side.
98. a rigid frame.
99. sideward protective extension in sitting.
100. 40 to 85%25 HR reserve (Karvonens formula).
101. independent in wheelchair mobility.
102. 9-11.
103. lower cervical flexion.
104. tibial, femoral, and pelvic internal rotation.
105. put a single line through the incorrect date, initial, then make and date the correction.
106. pulse duration.
107. Horners syndrome.
108. vestibulocerebellum.
109. peripheral arterial disease.
110. discontinue treatment and notify his physician immediately.
111. whirlpool with povidone-iodine.
112. sundowning behavior.
113. provide posterior directed resistance to the right ASIS during stance.
114. prone-on-elbow pushups.
115. Level 1.0 Receiving.
116. have the same therapist reassess the patients after 6 months.

07 Sullivan C
Answers 31

117. heart rate.


118. explain that this is normal and that the stepping was a newborn reflex that has gone away.
119. 35-44mm.
120. rearfoot varus posting only.
121. pronator teres syndrome.
122. treat the patient, and bill for the 20 minute session given.
123. large electrodes, widely spaced.
124. supine, trunk in midline with small pillow under the scapula, arm extended on supporting pillow, and a small
towel roll under the knee.
125. somatosensory integrity.
126. congestive heart failure.
127. HR reserve formula and Ratings of Perceived Exertion.
128. discontinue UBE exercise; use massage and active assistive ROM.
129. poor light adaptation.
130. sliding board.
131. lift the front casters and ascend in a wheelie position.
132. decreased bone density.
133. a Pancoast tumor.
134. atraumatic shoulder instability.
135. readjusting the harness and continuing with the treatment.
136. reposition the height of the shelf and items to below shoulder height
137. D2 flexion.
138. cushion heel with a rearfoot valgus post.
139. 2%2b pulses.
140. hepatitis B.
141. rheumatoid arthritis.
142. prosthetic knee set too far anterior to the TKA line.
143. contoured foam seat.
144. walker and one leg, tuck her pelvis by extending the upper trunk, and swing her other leg through.
145. involve him in goal setting and have him participate in structuring the training session.
146. the activities of the non-rehab group were not properly monitored and may account for these results.
147. lateral lean toward the right.
148. a solid ankle AFO.
149. delay in achieving developmental milestones.
150. transverse processes of T8.
151. thoracic outlet syndrome.
152. a vertebral artery test.
153. ambulation using bilateral AFOs and canes.
154. a series of letters traced on the hand.
155. light resistance to forward pelvic rotation during swing.
156. XII.
157. inferior wall.
158. all values are abnormal.
159. dermatitis.
160. surgical repair.
161. a circumducted gait.
162. icing and massage to the residual limb.
163. the validity of the study was threatened with the introduction of sampling bias.
164. left sidelying with the head of the bed in the flat position.
165. a lesion of the spinal accessory nerve.
166. low detection sensitivity with recording electrodes placed closely together.
167. left posterolateral medulla or pons.
168. drowsiness and muscle weakness.
169. neocerebellum.
170. higher heart rate and arterial blood pressure.
171. ice, elevation and a splint for the limb.
172. refer him to his physician as the therapist suspects a small stroke.
173. consume a carbohydrate before or during practice to avoid hypoglycemia.
174. supracondylar/suprapatellar suspension.

07 Sullivan C
32 Answers

175. instruct the PTA to have the patient sit down and utilize mental practice of the task.
176. lateral.
177. each side allotting five minutes for each section.
178. transcutaneous electrical stimulation.
179. vestibular deficiency.
180. loss of the hippocampus and declarative memory function.
181. 2 daily sessions of 30 minutes at 4070%25 VO2max
182. compress with one hand to a depth of to the depth of the chest at a rate of 100/min.
183. hypokalemia
184. coronary artery disease.
185. stand in front and slightly to the right side.
186. a ramp with a slope of 1:12 with a level landing at the top of at least 60 by 60 inches.
187. increasing the limits of stability and improving center of pressure alignment.
188. sinus rhythm with upsloping ST segment depression.
189. alternate the side of the wheelchair power control.
190. facet syndrome.
191. use job rotation during the workday.
192. 5-second ramp up, 5-second stimulation, 5-second ramp down.
193. provide consistent feedback using a blocked practice schedule.
194. anterior cerebral artery syndrome.
195. elevation-induced pallor and dependent redness with the extremity in the gravity dependent position.
196. pelvic floor and gentle abdominal exercises for the first 4-6 weeks.
197. depression.
198. adding horizontal grab bars positioned at 45 inches.
199. reduce peak rise of the pelvis.
200. thoracic postural syndrome.

07 Sullivan C

Potrebbero piacerti anche